Expected maximum number of unpaired socksHow to calculate the expected value when betting on which pairings will be selectedNumber of ways I can fit rectangles within a longer rectangleHow many strings contain every letter of the alphabet?a multiset where two consecutive numbers must be dissimilarLetter combinatorics and probabilitiesHow do I create a 4 on 4 tournament where every player plays with every other player an equal number of times?Socks on a clothing lineExpected “overlap” between permutations of a multisetSimilar sequences of coin flipsNumber of unique terms in the nth power of a sum of sinusoids

What is the most important source of natural gas? coal, oil or other?

Is floating in space similar to falling under gravity?

Logarithm of dependent variable is uniformly distributed. How to calculate a confidence interval for the mean?

Infinite Sequence based on Simple Rule

When do characters level up?

Integrating an absolute function using Mathematica

What are the benefits of cryosleep?

How bitcoin nodes update UTXO set when their latests blocks are replaced?

Identifying an object pointer by generating and using a unique ID

Why colon to denote that a value belongs to a type?

Smart people send dumb people to a new planet on a space craft that crashes into a body of water

What does it mean when you think without speaking?

What are these (utility?) boxes at the side of the house?

If a person had control of every single cell of their body, would they be able to transform into another creature?

I think I may have violated academic integrity last year - what should I do?

How to plot an unstable attractor?

Is this resistor leaking? If so, is it a concern?

How to prevent bad sectors?

Where is the logic in castrating fighters?

Is the first derivative operation on a signal a causal system?

Can you heal a summoned creature?

Why does the 6502 have the BIT instruction?

How did early x86 BIOS programmers manage to program full blown TUIs given very few bytes of ROM/EPROM?

Were pen cap holes designed to prevent death by suffocation if swallowed?



Expected maximum number of unpaired socks


How to calculate the expected value when betting on which pairings will be selectedNumber of ways I can fit rectangles within a longer rectangleHow many strings contain every letter of the alphabet?a multiset where two consecutive numbers must be dissimilarLetter combinatorics and probabilitiesHow do I create a 4 on 4 tournament where every player plays with every other player an equal number of times?Socks on a clothing lineExpected “overlap” between permutations of a multisetSimilar sequences of coin flipsNumber of unique terms in the nth power of a sum of sinusoids













41












$begingroup$


Like all combinatoric problems, this one is probably equivalent to another, well-known one, but I haven't managed to find such an equivalent problem (and OEIS didn't help), so I offer this one as being possibly new and possibly interesting.



Problem statement



I have $2N$ socks in a laundry basket, and I am hanging them on the hot pipes to dry. To make life easier later, I want to hang them in pairs. Since it is dark where the pipes are, I adopt the following algorithm:



  1. Take a sock at random from the basket.

  2. If it matches one that is already on my arm, hang them both on the pipes: the one in my hand and the matching one taken from my arm.

  3. If it does not match one that is already on my arm, hang it on my arm with the others.

  4. Do this $2N$ times.

The question is: How long does my arm have to be?



Clearly, the minimum length is $1$, for instance if the socks come out in the order $AABBCC$. Equally clearly, the maximum length is $N$, for instance if the socks come out as $ABCABC$. But what is the likeliest length? Or the average length? Or what sort of distribution do the required lengths have?



It turns out to be easiest to parameterise the results not by $2N$, the number of socks, but by $2N-1$, which I will call $M$.



The first few results



(Notation: $n!!$ is the semifactorial, the factorial including only odd numbers; thus $7!!=7times 5times 3times 1$).



In each case I provide the frequency for each possible arm length, starting with a length of 1. I use frequencies rather than probabilities because they are easier to type, but you can get the probabilities by dividing by $M!!$.



$$
beginarrayc
M \
hline
1 & 1 \
3 & 1 & 2 \
5 & 1 & 8 & 6 \
7 & 1 & 30 & 50 & 24 \
9 & 1 & 148 & 340 & 336 & 120 \
endarray
$$

It would be good to know (for example) if these frequencies tend to some sort of known distribution as $Mtoinfty$, just as the binomial coefficients do.



But, as I said at the beginning, this may just be a re-encoding of a known combinatorial problem, carrying a lot of previously worked out results along with it. I thought, for instance, of the lengths of random walks in $N$ dimensions with only one step forward and one step back being allowed in each dimension – but that looked too complicated to give any straightforward direction to follow.



Background: methods



In case it is interesting or helpful, I obtained the results above by means of a two-dimensional generating function, in which the coefficient of $y^n$ identified the arm length needed and the coefficient of $x^n$ identified how many socks had been retrieved at the [first] time that this length was reached. Calling the resulting generating function $A_M(x,y)$, the recurrence I used was:



$$A_M=MxyA_M-2+x^2(x-y)fracpartialpartial xA_M-2+(1-x^2)xy$$



which is based on sound first principles and matches the results of manual calculation up to $M=5$. Having found a polynomial, I substitute $x=1$ and the numbers in the table above are then the coefficients of the powers of $y$.



But, mathematics being close to comedy, all this elaboration may be an unnecessarily complicated way to get to a result too trivial to be found even in OEIS. Is it?










share|cite|improve this question









$endgroup$
















    41












    $begingroup$


    Like all combinatoric problems, this one is probably equivalent to another, well-known one, but I haven't managed to find such an equivalent problem (and OEIS didn't help), so I offer this one as being possibly new and possibly interesting.



    Problem statement



    I have $2N$ socks in a laundry basket, and I am hanging them on the hot pipes to dry. To make life easier later, I want to hang them in pairs. Since it is dark where the pipes are, I adopt the following algorithm:



    1. Take a sock at random from the basket.

    2. If it matches one that is already on my arm, hang them both on the pipes: the one in my hand and the matching one taken from my arm.

    3. If it does not match one that is already on my arm, hang it on my arm with the others.

    4. Do this $2N$ times.

    The question is: How long does my arm have to be?



    Clearly, the minimum length is $1$, for instance if the socks come out in the order $AABBCC$. Equally clearly, the maximum length is $N$, for instance if the socks come out as $ABCABC$. But what is the likeliest length? Or the average length? Or what sort of distribution do the required lengths have?



    It turns out to be easiest to parameterise the results not by $2N$, the number of socks, but by $2N-1$, which I will call $M$.



    The first few results



    (Notation: $n!!$ is the semifactorial, the factorial including only odd numbers; thus $7!!=7times 5times 3times 1$).



    In each case I provide the frequency for each possible arm length, starting with a length of 1. I use frequencies rather than probabilities because they are easier to type, but you can get the probabilities by dividing by $M!!$.



    $$
    beginarrayc
    M \
    hline
    1 & 1 \
    3 & 1 & 2 \
    5 & 1 & 8 & 6 \
    7 & 1 & 30 & 50 & 24 \
    9 & 1 & 148 & 340 & 336 & 120 \
    endarray
    $$

    It would be good to know (for example) if these frequencies tend to some sort of known distribution as $Mtoinfty$, just as the binomial coefficients do.



    But, as I said at the beginning, this may just be a re-encoding of a known combinatorial problem, carrying a lot of previously worked out results along with it. I thought, for instance, of the lengths of random walks in $N$ dimensions with only one step forward and one step back being allowed in each dimension – but that looked too complicated to give any straightforward direction to follow.



    Background: methods



    In case it is interesting or helpful, I obtained the results above by means of a two-dimensional generating function, in which the coefficient of $y^n$ identified the arm length needed and the coefficient of $x^n$ identified how many socks had been retrieved at the [first] time that this length was reached. Calling the resulting generating function $A_M(x,y)$, the recurrence I used was:



    $$A_M=MxyA_M-2+x^2(x-y)fracpartialpartial xA_M-2+(1-x^2)xy$$



    which is based on sound first principles and matches the results of manual calculation up to $M=5$. Having found a polynomial, I substitute $x=1$ and the numbers in the table above are then the coefficients of the powers of $y$.



    But, mathematics being close to comedy, all this elaboration may be an unnecessarily complicated way to get to a result too trivial to be found even in OEIS. Is it?










    share|cite|improve this question









    $endgroup$














      41












      41








      41


      14



      $begingroup$


      Like all combinatoric problems, this one is probably equivalent to another, well-known one, but I haven't managed to find such an equivalent problem (and OEIS didn't help), so I offer this one as being possibly new and possibly interesting.



      Problem statement



      I have $2N$ socks in a laundry basket, and I am hanging them on the hot pipes to dry. To make life easier later, I want to hang them in pairs. Since it is dark where the pipes are, I adopt the following algorithm:



      1. Take a sock at random from the basket.

      2. If it matches one that is already on my arm, hang them both on the pipes: the one in my hand and the matching one taken from my arm.

      3. If it does not match one that is already on my arm, hang it on my arm with the others.

      4. Do this $2N$ times.

      The question is: How long does my arm have to be?



      Clearly, the minimum length is $1$, for instance if the socks come out in the order $AABBCC$. Equally clearly, the maximum length is $N$, for instance if the socks come out as $ABCABC$. But what is the likeliest length? Or the average length? Or what sort of distribution do the required lengths have?



      It turns out to be easiest to parameterise the results not by $2N$, the number of socks, but by $2N-1$, which I will call $M$.



      The first few results



      (Notation: $n!!$ is the semifactorial, the factorial including only odd numbers; thus $7!!=7times 5times 3times 1$).



      In each case I provide the frequency for each possible arm length, starting with a length of 1. I use frequencies rather than probabilities because they are easier to type, but you can get the probabilities by dividing by $M!!$.



      $$
      beginarrayc
      M \
      hline
      1 & 1 \
      3 & 1 & 2 \
      5 & 1 & 8 & 6 \
      7 & 1 & 30 & 50 & 24 \
      9 & 1 & 148 & 340 & 336 & 120 \
      endarray
      $$

      It would be good to know (for example) if these frequencies tend to some sort of known distribution as $Mtoinfty$, just as the binomial coefficients do.



      But, as I said at the beginning, this may just be a re-encoding of a known combinatorial problem, carrying a lot of previously worked out results along with it. I thought, for instance, of the lengths of random walks in $N$ dimensions with only one step forward and one step back being allowed in each dimension – but that looked too complicated to give any straightforward direction to follow.



      Background: methods



      In case it is interesting or helpful, I obtained the results above by means of a two-dimensional generating function, in which the coefficient of $y^n$ identified the arm length needed and the coefficient of $x^n$ identified how many socks had been retrieved at the [first] time that this length was reached. Calling the resulting generating function $A_M(x,y)$, the recurrence I used was:



      $$A_M=MxyA_M-2+x^2(x-y)fracpartialpartial xA_M-2+(1-x^2)xy$$



      which is based on sound first principles and matches the results of manual calculation up to $M=5$. Having found a polynomial, I substitute $x=1$ and the numbers in the table above are then the coefficients of the powers of $y$.



      But, mathematics being close to comedy, all this elaboration may be an unnecessarily complicated way to get to a result too trivial to be found even in OEIS. Is it?










      share|cite|improve this question









      $endgroup$




      Like all combinatoric problems, this one is probably equivalent to another, well-known one, but I haven't managed to find such an equivalent problem (and OEIS didn't help), so I offer this one as being possibly new and possibly interesting.



      Problem statement



      I have $2N$ socks in a laundry basket, and I am hanging them on the hot pipes to dry. To make life easier later, I want to hang them in pairs. Since it is dark where the pipes are, I adopt the following algorithm:



      1. Take a sock at random from the basket.

      2. If it matches one that is already on my arm, hang them both on the pipes: the one in my hand and the matching one taken from my arm.

      3. If it does not match one that is already on my arm, hang it on my arm with the others.

      4. Do this $2N$ times.

      The question is: How long does my arm have to be?



      Clearly, the minimum length is $1$, for instance if the socks come out in the order $AABBCC$. Equally clearly, the maximum length is $N$, for instance if the socks come out as $ABCABC$. But what is the likeliest length? Or the average length? Or what sort of distribution do the required lengths have?



      It turns out to be easiest to parameterise the results not by $2N$, the number of socks, but by $2N-1$, which I will call $M$.



      The first few results



      (Notation: $n!!$ is the semifactorial, the factorial including only odd numbers; thus $7!!=7times 5times 3times 1$).



      In each case I provide the frequency for each possible arm length, starting with a length of 1. I use frequencies rather than probabilities because they are easier to type, but you can get the probabilities by dividing by $M!!$.



      $$
      beginarrayc
      M \
      hline
      1 & 1 \
      3 & 1 & 2 \
      5 & 1 & 8 & 6 \
      7 & 1 & 30 & 50 & 24 \
      9 & 1 & 148 & 340 & 336 & 120 \
      endarray
      $$

      It would be good to know (for example) if these frequencies tend to some sort of known distribution as $Mtoinfty$, just as the binomial coefficients do.



      But, as I said at the beginning, this may just be a re-encoding of a known combinatorial problem, carrying a lot of previously worked out results along with it. I thought, for instance, of the lengths of random walks in $N$ dimensions with only one step forward and one step back being allowed in each dimension – but that looked too complicated to give any straightforward direction to follow.



      Background: methods



      In case it is interesting or helpful, I obtained the results above by means of a two-dimensional generating function, in which the coefficient of $y^n$ identified the arm length needed and the coefficient of $x^n$ identified how many socks had been retrieved at the [first] time that this length was reached. Calling the resulting generating function $A_M(x,y)$, the recurrence I used was:



      $$A_M=MxyA_M-2+x^2(x-y)fracpartialpartial xA_M-2+(1-x^2)xy$$



      which is based on sound first principles and matches the results of manual calculation up to $M=5$. Having found a polynomial, I substitute $x=1$ and the numbers in the table above are then the coefficients of the powers of $y$.



      But, mathematics being close to comedy, all this elaboration may be an unnecessarily complicated way to get to a result too trivial to be found even in OEIS. Is it?







      combinatorics generating-functions






      share|cite|improve this question













      share|cite|improve this question











      share|cite|improve this question




      share|cite|improve this question










      asked May 21 at 10:12









      Martin KochanskiMartin Kochanski

      2,174411




      2,174411




















          3 Answers
          3






          active

          oldest

          votes


















          24












          $begingroup$

          I did some Monte Carlo with this interesting problem and came to some interesting conclusions. If you have $N$ pairs of socks the expected maximum arm length is slightly above $N/2$.



          First, I made 1,000,000 experiments with 100 pairs of socks and recorded maximum arm length reached in each one. For example, maximum arm length of 54 was reached about 90,000 times. And it all looks like a normal distribution to me. The average value of maximum arm length was 53.91, confirmed several times in a row.



          enter image description here



          Nothing changed with 100 pairs of socks and 10,000,000 experiments. Average value remained the same. So it looks like you need about a million runs to draw up a meaningful conclusion.



          enter image description here



          Here is what I got when I doubled the number of socks to 200 pairs. Maximum arm length on average was 105.12, still above 50%. I got the same value in several repeated experiments ($pm0.01$).



          enter image description here



          Finally, I decided to check expected maximum arm length for different number of sock pairs, from 10 to 250. Each number of pairs was tested 2,000,000 times before the average value was calculated. Here are the results:



          $$
          beginarrayrr
          textbfPairs & textbfArm Length & textbfIncrement \
          hline
          10 & 6.49 & \
          20 & 12.03 & 5.54 \
          30 & 17.41 & 5.38 \
          40 & 22.71 & 5.30 \
          50 & 27.97 & 5.26 \
          60 & 33.20 & 5.23 \
          70 & 38.40 & 5.20 \
          80 & 43.59 & 5.19 \
          90 & 48.75 & 5.16 \
          100 & 53.91 & 5.16 \
          110 & 59.07 & 5.16 \
          120 & 64.20 & 5.13 \
          130 & 69.33 & 5.13 \
          140 & 74.46 & 5.13 \
          150 & 79.58 & 5.12 \
          160 & 84.69 & 5.11 \
          170 & 89.80 & 5.11 \
          180 & 94.91 & 5.11 \
          190 & 100.02 & 5.11 \
          200 & 105.11 & 5.09 \
          210 & 110.20 & 5.09 \
          220 & 115.29 & 5.09 \
          230 & 120.38 & 5.09 \
          240 & 125.47 & 5.09 \
          250 & 130.56 & 5.09
          endarray
          $$



          enter image description here



          It looks like a straight line but it's actually an arc, slightly bended downwards (take a look at the increment column).



          Finally, here is the Java code that I used for my experiments.



          import java.util.ArrayList;
          import java.util.Collections;
          import java.util.HashSet;
          import java.util.List;
          import java.util.Set;

          public class Basket
          public static final int PAIRS = 250;
          public static final int NUM_EXPERIMENTS = 2_000_000;

          int n;
          List<Integer> basket;
          Set<Integer> arm;

          public Basket(int n)
          // basket size
          this.n = n;
          // socks are here
          this.basket = new ArrayList<Integer>();
          // arm is just a set of different socks
          this.arm = new HashSet<Integer>();
          // add a pair of same socks to the basket
          for(int i = 0; i < n; i++)
          basket.add(i);
          basket.add(i);

          // shuffle the basket
          Collections.shuffle(basket);


          // returns maximum arm length
          int hangSocks()
          // maximum arm length
          int maxArmLength = 0;
          // we have to hang all socks
          for(int i = 0; i < 2 * n; i++)
          // take one sock from the basket
          int sock = basket.get(i);
          // if the sock of the same color is already on your arm...
          if(arm.contains(sock))
          // ...remove sock from your arm and put the pair over the hot pipe
          arm.remove(sock);

          else
          // put the sock on your arm
          arm.add(sock);
          // update maximum arm length
          maxArmLength = Math.max(maxArmLength, arm.size());


          return maxArmLength;


          public static void main(String[] args)
          // results of our experiments will be stored here
          int[] results = new int[PAIRS + 1];
          // run millions of experiments
          for(int i = 0; i < NUM_EXPERIMENTS; i++)
          Basket b = new Basket(PAIRS);
          // arm length in a single experiment
          int length = b.hangSocks();
          // remember how often this result appeared
          results[length]++;

          // print results in CSV format so that we can plot them in Excel
          for(int i = 0; i < results.length; i++)
          System.out.println(i + "," + results[i]);

          // find average arm length
          int sum = 0;
          for(int i = 0; i < results.length; i++)
          sum += i * results[i];

          double average = (double) sum / (double) NUM_EXPERIMENTS;
          System.out.println(String.format("Average arm length is %.2f", average));





          EDIT: For N=500, the average value of maximum arm length after 2,000,000 tests is 257.19. For N=1000, the result is 509.23.



          It seems that for $Ntoinfty$, the result goes down to $N/2$. I don't know how to prove this.






          share|cite|improve this answer











          $endgroup$












          • $begingroup$
            Splendid. There is just one snag (or one advantage!): these figures contradict mine quite strongly. I can't verifiably take my recurrence as far up as your numbers, so I wonder: could you let me know what (a) your average maximum arm length and (b) your commonest maximum arm length are for each of 1 to 9 pairs? With any luck we will get a clear enough and noticeable enough deviation for me to identify where I've been going wrong. I am only asking because you have already written the program and plugging in some small numbers shouldn't be any work.
            $endgroup$
            – Martin Kochanski
            May 21 at 17:57










          • $begingroup$
            @MartinKochanski I'll get back to you tomorrow.
            $endgroup$
            – Oldboy
            May 21 at 19:45










          • $begingroup$
            Thanks: I have just done some more working out, and I can see where my recurrence went wrong. So don't bother with any more simulations after all.
            $endgroup$
            – Martin Kochanski
            May 21 at 20:01


















          14












          $begingroup$

          The expected number of single socks is maximized when you are halfway through. When you have drawn $N$ socks the chance that a given pair has one on your arm is $frac 2N^22N^2+2N(N-1)=fracN^22N^2-Napprox frac 12+frac 12N$. If we make the socks distinguishable, to have one on your arm sock $1$ of a pair has $2N$ positions it can be in, then sock $2$ has $N$ choices-to be in the other half of the run. To not have one on your arm sock $1$ again has $2N$ choices but sock $2$ has only $N-1$ as it must be in the same half of the run. This says the expected number on your arm is $frac N^22N-1approx frac N+12$.



          The expected value being below the mode of Oldboy's distributions says that the distribution is not symmetric around the mode.



          Note that this addresses the expected maximum at a given point. The expected maximum over a distribution can be higher as Empy2 explains.






          share|cite|improve this answer











          $endgroup$












          • $begingroup$
            Could you please explain why when you have drawn $N$ socks the chance that a given pair has one on your arm is $frac N^2N^2+N(N-1)$?
            $endgroup$
            – Hans
            May 23 at 5:47










          • $begingroup$
            @Hans: I said the expected value of the number of socks on your arm was that. I computed the chance you have one of a particular pair on your arm, then multiplied by $N$ using the linearity of expectation to get the total expectation.
            $endgroup$
            – Ross Millikan
            May 23 at 14:36











          • $begingroup$
            I am confused. I just copied your claim: "When you have drawn $N$ socks the chance that a given pair has one on your arm is $frac N^2N^2+N(N-1)$." Are you saying you do not mean this expression to be a probability but an expectation? If you do mean this to be a probability, I do not understand your rationale for this expression. In fact, I got a different expression. Could you please derive this expression of the probability?
            $endgroup$
            – Hans
            May 23 at 17:51










          • $begingroup$
            @Hans: For one pair, the probability and the expectation are the same because you either have one sock on your arm or not. I justify the probability in the next sentence, though I left out a factor $2$. There are $2N^2$ ways to choose the positions of the socks of one pair so you have one on your arm at the midpoint and $2N(N-1)$ ways to not have one on your arm.
            $endgroup$
            – Ross Millikan
            May 23 at 19:35










          • $begingroup$
            You are right. +1. I took the individual socks of a pair as indistinguishable which was wrong.
            $endgroup$
            – Hans
            2 days ago


















          8












          $begingroup$

          The extra bits, above $N/2$ in Oldboy's table, are near $sqrt[3]N$. I have some ideas why that might be true.



          First, the expected number of socks on the arm at the $N+x$th sock is $(N^2-x^2)/(2N-1)$.



          Near the $N$th sock, the number of socks on the arm follows a random walk. It is symmetric at the $N$th sock, but has a negative bias of $x^2/2N$ at the $N+x$th sock.

          Around the $N+yN^2/3$th sock, a symmetric random walk would have moved $O(sqrtyN^2/3)$, but the negative bias is also $O((yN^2/3)^2/N)$, so both are $O(N^1/3)$. The negative bias will dominate for large $y$, so the maximum value will be in this domain. The random variation dominates for small $y$.



          So the maximum is likely to be $N/2+O(N^1/3)$.



          EDIT: the variance in the number of socks on the arm at the $N$th sock is $$frac2N^2(N-1)^2(2N-1)^2(2N-3)approxfrac N4$$
          So the width of the bell curve in Oldboy's graphs is roughly $sqrtN$. But this effect is symmetric above and below the mean $N^2/(2N-1)$. The maximum of the random walk is not symmetric, and shifts the bell curve to the right, but that effect $(O(N^1/3))$ is smaller than the variation from one laundry basket to the next $(O(N^1/2))$






          share|cite|improve this answer











          $endgroup$












          • $begingroup$
            +1 Excellent obeservation and it fits my data almost perfectly.
            $endgroup$
            – Oldboy
            May 22 at 6:59










          • $begingroup$
            The denominator of the variance approximation of the number of socks on the arm should be $4$ not $2$.
            $endgroup$
            – Hans
            7 hours ago










          • $begingroup$
            1) By the random walk of the number of socks on the arm "has a negative bias", do you simply mean the difference between the mean and $frac N2$ which is $-fracx^22N-1$? 2) Could you please explain your statement "a symmetric random walk would have moved $O(sqrtyN^2/3)$"? Should the standard deviation be instead $sqrtN+yN^2/3$?
            $endgroup$
            – Hans
            2 hours ago











          • $begingroup$
            I meant that the middle, at $N$, is expected to be the highest point, butas you wander away from there the is a random walk that goes above and below, so the maximum is likely to be above the centre point. It will probably only get $O(N^1/3)$ above before the mean drags it back below the centre point
            $endgroup$
            – Empy2
            2 hours ago











          Your Answer








          StackExchange.ready(function()
          var channelOptions =
          tags: "".split(" "),
          id: "69"
          ;
          initTagRenderer("".split(" "), "".split(" "), channelOptions);

          StackExchange.using("externalEditor", function()
          // Have to fire editor after snippets, if snippets enabled
          if (StackExchange.settings.snippets.snippetsEnabled)
          StackExchange.using("snippets", function()
          createEditor();
          );

          else
          createEditor();

          );

          function createEditor()
          StackExchange.prepareEditor(
          heartbeatType: 'answer',
          autoActivateHeartbeat: false,
          convertImagesToLinks: true,
          noModals: true,
          showLowRepImageUploadWarning: true,
          reputationToPostImages: 10,
          bindNavPrevention: true,
          postfix: "",
          imageUploader:
          brandingHtml: "Powered by u003ca class="icon-imgur-white" href="https://imgur.com/"u003eu003c/au003e",
          contentPolicyHtml: "User contributions licensed under u003ca href="https://creativecommons.org/licenses/by-sa/3.0/"u003ecc by-sa 3.0 with attribution requiredu003c/au003e u003ca href="https://stackoverflow.com/legal/content-policy"u003e(content policy)u003c/au003e",
          allowUrls: true
          ,
          noCode: true, onDemand: true,
          discardSelector: ".discard-answer"
          ,immediatelyShowMarkdownHelp:true
          );



          );













          draft saved

          draft discarded


















          StackExchange.ready(
          function ()
          StackExchange.openid.initPostLogin('.new-post-login', 'https%3a%2f%2fmath.stackexchange.com%2fquestions%2f3234157%2fexpected-maximum-number-of-unpaired-socks%23new-answer', 'question_page');

          );

          Post as a guest















          Required, but never shown

























          3 Answers
          3






          active

          oldest

          votes








          3 Answers
          3






          active

          oldest

          votes









          active

          oldest

          votes






          active

          oldest

          votes









          24












          $begingroup$

          I did some Monte Carlo with this interesting problem and came to some interesting conclusions. If you have $N$ pairs of socks the expected maximum arm length is slightly above $N/2$.



          First, I made 1,000,000 experiments with 100 pairs of socks and recorded maximum arm length reached in each one. For example, maximum arm length of 54 was reached about 90,000 times. And it all looks like a normal distribution to me. The average value of maximum arm length was 53.91, confirmed several times in a row.



          enter image description here



          Nothing changed with 100 pairs of socks and 10,000,000 experiments. Average value remained the same. So it looks like you need about a million runs to draw up a meaningful conclusion.



          enter image description here



          Here is what I got when I doubled the number of socks to 200 pairs. Maximum arm length on average was 105.12, still above 50%. I got the same value in several repeated experiments ($pm0.01$).



          enter image description here



          Finally, I decided to check expected maximum arm length for different number of sock pairs, from 10 to 250. Each number of pairs was tested 2,000,000 times before the average value was calculated. Here are the results:



          $$
          beginarrayrr
          textbfPairs & textbfArm Length & textbfIncrement \
          hline
          10 & 6.49 & \
          20 & 12.03 & 5.54 \
          30 & 17.41 & 5.38 \
          40 & 22.71 & 5.30 \
          50 & 27.97 & 5.26 \
          60 & 33.20 & 5.23 \
          70 & 38.40 & 5.20 \
          80 & 43.59 & 5.19 \
          90 & 48.75 & 5.16 \
          100 & 53.91 & 5.16 \
          110 & 59.07 & 5.16 \
          120 & 64.20 & 5.13 \
          130 & 69.33 & 5.13 \
          140 & 74.46 & 5.13 \
          150 & 79.58 & 5.12 \
          160 & 84.69 & 5.11 \
          170 & 89.80 & 5.11 \
          180 & 94.91 & 5.11 \
          190 & 100.02 & 5.11 \
          200 & 105.11 & 5.09 \
          210 & 110.20 & 5.09 \
          220 & 115.29 & 5.09 \
          230 & 120.38 & 5.09 \
          240 & 125.47 & 5.09 \
          250 & 130.56 & 5.09
          endarray
          $$



          enter image description here



          It looks like a straight line but it's actually an arc, slightly bended downwards (take a look at the increment column).



          Finally, here is the Java code that I used for my experiments.



          import java.util.ArrayList;
          import java.util.Collections;
          import java.util.HashSet;
          import java.util.List;
          import java.util.Set;

          public class Basket
          public static final int PAIRS = 250;
          public static final int NUM_EXPERIMENTS = 2_000_000;

          int n;
          List<Integer> basket;
          Set<Integer> arm;

          public Basket(int n)
          // basket size
          this.n = n;
          // socks are here
          this.basket = new ArrayList<Integer>();
          // arm is just a set of different socks
          this.arm = new HashSet<Integer>();
          // add a pair of same socks to the basket
          for(int i = 0; i < n; i++)
          basket.add(i);
          basket.add(i);

          // shuffle the basket
          Collections.shuffle(basket);


          // returns maximum arm length
          int hangSocks()
          // maximum arm length
          int maxArmLength = 0;
          // we have to hang all socks
          for(int i = 0; i < 2 * n; i++)
          // take one sock from the basket
          int sock = basket.get(i);
          // if the sock of the same color is already on your arm...
          if(arm.contains(sock))
          // ...remove sock from your arm and put the pair over the hot pipe
          arm.remove(sock);

          else
          // put the sock on your arm
          arm.add(sock);
          // update maximum arm length
          maxArmLength = Math.max(maxArmLength, arm.size());


          return maxArmLength;


          public static void main(String[] args)
          // results of our experiments will be stored here
          int[] results = new int[PAIRS + 1];
          // run millions of experiments
          for(int i = 0; i < NUM_EXPERIMENTS; i++)
          Basket b = new Basket(PAIRS);
          // arm length in a single experiment
          int length = b.hangSocks();
          // remember how often this result appeared
          results[length]++;

          // print results in CSV format so that we can plot them in Excel
          for(int i = 0; i < results.length; i++)
          System.out.println(i + "," + results[i]);

          // find average arm length
          int sum = 0;
          for(int i = 0; i < results.length; i++)
          sum += i * results[i];

          double average = (double) sum / (double) NUM_EXPERIMENTS;
          System.out.println(String.format("Average arm length is %.2f", average));





          EDIT: For N=500, the average value of maximum arm length after 2,000,000 tests is 257.19. For N=1000, the result is 509.23.



          It seems that for $Ntoinfty$, the result goes down to $N/2$. I don't know how to prove this.






          share|cite|improve this answer











          $endgroup$












          • $begingroup$
            Splendid. There is just one snag (or one advantage!): these figures contradict mine quite strongly. I can't verifiably take my recurrence as far up as your numbers, so I wonder: could you let me know what (a) your average maximum arm length and (b) your commonest maximum arm length are for each of 1 to 9 pairs? With any luck we will get a clear enough and noticeable enough deviation for me to identify where I've been going wrong. I am only asking because you have already written the program and plugging in some small numbers shouldn't be any work.
            $endgroup$
            – Martin Kochanski
            May 21 at 17:57










          • $begingroup$
            @MartinKochanski I'll get back to you tomorrow.
            $endgroup$
            – Oldboy
            May 21 at 19:45










          • $begingroup$
            Thanks: I have just done some more working out, and I can see where my recurrence went wrong. So don't bother with any more simulations after all.
            $endgroup$
            – Martin Kochanski
            May 21 at 20:01















          24












          $begingroup$

          I did some Monte Carlo with this interesting problem and came to some interesting conclusions. If you have $N$ pairs of socks the expected maximum arm length is slightly above $N/2$.



          First, I made 1,000,000 experiments with 100 pairs of socks and recorded maximum arm length reached in each one. For example, maximum arm length of 54 was reached about 90,000 times. And it all looks like a normal distribution to me. The average value of maximum arm length was 53.91, confirmed several times in a row.



          enter image description here



          Nothing changed with 100 pairs of socks and 10,000,000 experiments. Average value remained the same. So it looks like you need about a million runs to draw up a meaningful conclusion.



          enter image description here



          Here is what I got when I doubled the number of socks to 200 pairs. Maximum arm length on average was 105.12, still above 50%. I got the same value in several repeated experiments ($pm0.01$).



          enter image description here



          Finally, I decided to check expected maximum arm length for different number of sock pairs, from 10 to 250. Each number of pairs was tested 2,000,000 times before the average value was calculated. Here are the results:



          $$
          beginarrayrr
          textbfPairs & textbfArm Length & textbfIncrement \
          hline
          10 & 6.49 & \
          20 & 12.03 & 5.54 \
          30 & 17.41 & 5.38 \
          40 & 22.71 & 5.30 \
          50 & 27.97 & 5.26 \
          60 & 33.20 & 5.23 \
          70 & 38.40 & 5.20 \
          80 & 43.59 & 5.19 \
          90 & 48.75 & 5.16 \
          100 & 53.91 & 5.16 \
          110 & 59.07 & 5.16 \
          120 & 64.20 & 5.13 \
          130 & 69.33 & 5.13 \
          140 & 74.46 & 5.13 \
          150 & 79.58 & 5.12 \
          160 & 84.69 & 5.11 \
          170 & 89.80 & 5.11 \
          180 & 94.91 & 5.11 \
          190 & 100.02 & 5.11 \
          200 & 105.11 & 5.09 \
          210 & 110.20 & 5.09 \
          220 & 115.29 & 5.09 \
          230 & 120.38 & 5.09 \
          240 & 125.47 & 5.09 \
          250 & 130.56 & 5.09
          endarray
          $$



          enter image description here



          It looks like a straight line but it's actually an arc, slightly bended downwards (take a look at the increment column).



          Finally, here is the Java code that I used for my experiments.



          import java.util.ArrayList;
          import java.util.Collections;
          import java.util.HashSet;
          import java.util.List;
          import java.util.Set;

          public class Basket
          public static final int PAIRS = 250;
          public static final int NUM_EXPERIMENTS = 2_000_000;

          int n;
          List<Integer> basket;
          Set<Integer> arm;

          public Basket(int n)
          // basket size
          this.n = n;
          // socks are here
          this.basket = new ArrayList<Integer>();
          // arm is just a set of different socks
          this.arm = new HashSet<Integer>();
          // add a pair of same socks to the basket
          for(int i = 0; i < n; i++)
          basket.add(i);
          basket.add(i);

          // shuffle the basket
          Collections.shuffle(basket);


          // returns maximum arm length
          int hangSocks()
          // maximum arm length
          int maxArmLength = 0;
          // we have to hang all socks
          for(int i = 0; i < 2 * n; i++)
          // take one sock from the basket
          int sock = basket.get(i);
          // if the sock of the same color is already on your arm...
          if(arm.contains(sock))
          // ...remove sock from your arm and put the pair over the hot pipe
          arm.remove(sock);

          else
          // put the sock on your arm
          arm.add(sock);
          // update maximum arm length
          maxArmLength = Math.max(maxArmLength, arm.size());


          return maxArmLength;


          public static void main(String[] args)
          // results of our experiments will be stored here
          int[] results = new int[PAIRS + 1];
          // run millions of experiments
          for(int i = 0; i < NUM_EXPERIMENTS; i++)
          Basket b = new Basket(PAIRS);
          // arm length in a single experiment
          int length = b.hangSocks();
          // remember how often this result appeared
          results[length]++;

          // print results in CSV format so that we can plot them in Excel
          for(int i = 0; i < results.length; i++)
          System.out.println(i + "," + results[i]);

          // find average arm length
          int sum = 0;
          for(int i = 0; i < results.length; i++)
          sum += i * results[i];

          double average = (double) sum / (double) NUM_EXPERIMENTS;
          System.out.println(String.format("Average arm length is %.2f", average));





          EDIT: For N=500, the average value of maximum arm length after 2,000,000 tests is 257.19. For N=1000, the result is 509.23.



          It seems that for $Ntoinfty$, the result goes down to $N/2$. I don't know how to prove this.






          share|cite|improve this answer











          $endgroup$












          • $begingroup$
            Splendid. There is just one snag (or one advantage!): these figures contradict mine quite strongly. I can't verifiably take my recurrence as far up as your numbers, so I wonder: could you let me know what (a) your average maximum arm length and (b) your commonest maximum arm length are for each of 1 to 9 pairs? With any luck we will get a clear enough and noticeable enough deviation for me to identify where I've been going wrong. I am only asking because you have already written the program and plugging in some small numbers shouldn't be any work.
            $endgroup$
            – Martin Kochanski
            May 21 at 17:57










          • $begingroup$
            @MartinKochanski I'll get back to you tomorrow.
            $endgroup$
            – Oldboy
            May 21 at 19:45










          • $begingroup$
            Thanks: I have just done some more working out, and I can see where my recurrence went wrong. So don't bother with any more simulations after all.
            $endgroup$
            – Martin Kochanski
            May 21 at 20:01













          24












          24








          24





          $begingroup$

          I did some Monte Carlo with this interesting problem and came to some interesting conclusions. If you have $N$ pairs of socks the expected maximum arm length is slightly above $N/2$.



          First, I made 1,000,000 experiments with 100 pairs of socks and recorded maximum arm length reached in each one. For example, maximum arm length of 54 was reached about 90,000 times. And it all looks like a normal distribution to me. The average value of maximum arm length was 53.91, confirmed several times in a row.



          enter image description here



          Nothing changed with 100 pairs of socks and 10,000,000 experiments. Average value remained the same. So it looks like you need about a million runs to draw up a meaningful conclusion.



          enter image description here



          Here is what I got when I doubled the number of socks to 200 pairs. Maximum arm length on average was 105.12, still above 50%. I got the same value in several repeated experiments ($pm0.01$).



          enter image description here



          Finally, I decided to check expected maximum arm length for different number of sock pairs, from 10 to 250. Each number of pairs was tested 2,000,000 times before the average value was calculated. Here are the results:



          $$
          beginarrayrr
          textbfPairs & textbfArm Length & textbfIncrement \
          hline
          10 & 6.49 & \
          20 & 12.03 & 5.54 \
          30 & 17.41 & 5.38 \
          40 & 22.71 & 5.30 \
          50 & 27.97 & 5.26 \
          60 & 33.20 & 5.23 \
          70 & 38.40 & 5.20 \
          80 & 43.59 & 5.19 \
          90 & 48.75 & 5.16 \
          100 & 53.91 & 5.16 \
          110 & 59.07 & 5.16 \
          120 & 64.20 & 5.13 \
          130 & 69.33 & 5.13 \
          140 & 74.46 & 5.13 \
          150 & 79.58 & 5.12 \
          160 & 84.69 & 5.11 \
          170 & 89.80 & 5.11 \
          180 & 94.91 & 5.11 \
          190 & 100.02 & 5.11 \
          200 & 105.11 & 5.09 \
          210 & 110.20 & 5.09 \
          220 & 115.29 & 5.09 \
          230 & 120.38 & 5.09 \
          240 & 125.47 & 5.09 \
          250 & 130.56 & 5.09
          endarray
          $$



          enter image description here



          It looks like a straight line but it's actually an arc, slightly bended downwards (take a look at the increment column).



          Finally, here is the Java code that I used for my experiments.



          import java.util.ArrayList;
          import java.util.Collections;
          import java.util.HashSet;
          import java.util.List;
          import java.util.Set;

          public class Basket
          public static final int PAIRS = 250;
          public static final int NUM_EXPERIMENTS = 2_000_000;

          int n;
          List<Integer> basket;
          Set<Integer> arm;

          public Basket(int n)
          // basket size
          this.n = n;
          // socks are here
          this.basket = new ArrayList<Integer>();
          // arm is just a set of different socks
          this.arm = new HashSet<Integer>();
          // add a pair of same socks to the basket
          for(int i = 0; i < n; i++)
          basket.add(i);
          basket.add(i);

          // shuffle the basket
          Collections.shuffle(basket);


          // returns maximum arm length
          int hangSocks()
          // maximum arm length
          int maxArmLength = 0;
          // we have to hang all socks
          for(int i = 0; i < 2 * n; i++)
          // take one sock from the basket
          int sock = basket.get(i);
          // if the sock of the same color is already on your arm...
          if(arm.contains(sock))
          // ...remove sock from your arm and put the pair over the hot pipe
          arm.remove(sock);

          else
          // put the sock on your arm
          arm.add(sock);
          // update maximum arm length
          maxArmLength = Math.max(maxArmLength, arm.size());


          return maxArmLength;


          public static void main(String[] args)
          // results of our experiments will be stored here
          int[] results = new int[PAIRS + 1];
          // run millions of experiments
          for(int i = 0; i < NUM_EXPERIMENTS; i++)
          Basket b = new Basket(PAIRS);
          // arm length in a single experiment
          int length = b.hangSocks();
          // remember how often this result appeared
          results[length]++;

          // print results in CSV format so that we can plot them in Excel
          for(int i = 0; i < results.length; i++)
          System.out.println(i + "," + results[i]);

          // find average arm length
          int sum = 0;
          for(int i = 0; i < results.length; i++)
          sum += i * results[i];

          double average = (double) sum / (double) NUM_EXPERIMENTS;
          System.out.println(String.format("Average arm length is %.2f", average));





          EDIT: For N=500, the average value of maximum arm length after 2,000,000 tests is 257.19. For N=1000, the result is 509.23.



          It seems that for $Ntoinfty$, the result goes down to $N/2$. I don't know how to prove this.






          share|cite|improve this answer











          $endgroup$



          I did some Monte Carlo with this interesting problem and came to some interesting conclusions. If you have $N$ pairs of socks the expected maximum arm length is slightly above $N/2$.



          First, I made 1,000,000 experiments with 100 pairs of socks and recorded maximum arm length reached in each one. For example, maximum arm length of 54 was reached about 90,000 times. And it all looks like a normal distribution to me. The average value of maximum arm length was 53.91, confirmed several times in a row.



          enter image description here



          Nothing changed with 100 pairs of socks and 10,000,000 experiments. Average value remained the same. So it looks like you need about a million runs to draw up a meaningful conclusion.



          enter image description here



          Here is what I got when I doubled the number of socks to 200 pairs. Maximum arm length on average was 105.12, still above 50%. I got the same value in several repeated experiments ($pm0.01$).



          enter image description here



          Finally, I decided to check expected maximum arm length for different number of sock pairs, from 10 to 250. Each number of pairs was tested 2,000,000 times before the average value was calculated. Here are the results:



          $$
          beginarrayrr
          textbfPairs & textbfArm Length & textbfIncrement \
          hline
          10 & 6.49 & \
          20 & 12.03 & 5.54 \
          30 & 17.41 & 5.38 \
          40 & 22.71 & 5.30 \
          50 & 27.97 & 5.26 \
          60 & 33.20 & 5.23 \
          70 & 38.40 & 5.20 \
          80 & 43.59 & 5.19 \
          90 & 48.75 & 5.16 \
          100 & 53.91 & 5.16 \
          110 & 59.07 & 5.16 \
          120 & 64.20 & 5.13 \
          130 & 69.33 & 5.13 \
          140 & 74.46 & 5.13 \
          150 & 79.58 & 5.12 \
          160 & 84.69 & 5.11 \
          170 & 89.80 & 5.11 \
          180 & 94.91 & 5.11 \
          190 & 100.02 & 5.11 \
          200 & 105.11 & 5.09 \
          210 & 110.20 & 5.09 \
          220 & 115.29 & 5.09 \
          230 & 120.38 & 5.09 \
          240 & 125.47 & 5.09 \
          250 & 130.56 & 5.09
          endarray
          $$



          enter image description here



          It looks like a straight line but it's actually an arc, slightly bended downwards (take a look at the increment column).



          Finally, here is the Java code that I used for my experiments.



          import java.util.ArrayList;
          import java.util.Collections;
          import java.util.HashSet;
          import java.util.List;
          import java.util.Set;

          public class Basket
          public static final int PAIRS = 250;
          public static final int NUM_EXPERIMENTS = 2_000_000;

          int n;
          List<Integer> basket;
          Set<Integer> arm;

          public Basket(int n)
          // basket size
          this.n = n;
          // socks are here
          this.basket = new ArrayList<Integer>();
          // arm is just a set of different socks
          this.arm = new HashSet<Integer>();
          // add a pair of same socks to the basket
          for(int i = 0; i < n; i++)
          basket.add(i);
          basket.add(i);

          // shuffle the basket
          Collections.shuffle(basket);


          // returns maximum arm length
          int hangSocks()
          // maximum arm length
          int maxArmLength = 0;
          // we have to hang all socks
          for(int i = 0; i < 2 * n; i++)
          // take one sock from the basket
          int sock = basket.get(i);
          // if the sock of the same color is already on your arm...
          if(arm.contains(sock))
          // ...remove sock from your arm and put the pair over the hot pipe
          arm.remove(sock);

          else
          // put the sock on your arm
          arm.add(sock);
          // update maximum arm length
          maxArmLength = Math.max(maxArmLength, arm.size());


          return maxArmLength;


          public static void main(String[] args)
          // results of our experiments will be stored here
          int[] results = new int[PAIRS + 1];
          // run millions of experiments
          for(int i = 0; i < NUM_EXPERIMENTS; i++)
          Basket b = new Basket(PAIRS);
          // arm length in a single experiment
          int length = b.hangSocks();
          // remember how often this result appeared
          results[length]++;

          // print results in CSV format so that we can plot them in Excel
          for(int i = 0; i < results.length; i++)
          System.out.println(i + "," + results[i]);

          // find average arm length
          int sum = 0;
          for(int i = 0; i < results.length; i++)
          sum += i * results[i];

          double average = (double) sum / (double) NUM_EXPERIMENTS;
          System.out.println(String.format("Average arm length is %.2f", average));





          EDIT: For N=500, the average value of maximum arm length after 2,000,000 tests is 257.19. For N=1000, the result is 509.23.



          It seems that for $Ntoinfty$, the result goes down to $N/2$. I don't know how to prove this.







          share|cite|improve this answer














          share|cite|improve this answer



          share|cite|improve this answer








          edited May 21 at 13:43

























          answered May 21 at 13:08









          OldboyOldboy

          10.6k11442




          10.6k11442











          • $begingroup$
            Splendid. There is just one snag (or one advantage!): these figures contradict mine quite strongly. I can't verifiably take my recurrence as far up as your numbers, so I wonder: could you let me know what (a) your average maximum arm length and (b) your commonest maximum arm length are for each of 1 to 9 pairs? With any luck we will get a clear enough and noticeable enough deviation for me to identify where I've been going wrong. I am only asking because you have already written the program and plugging in some small numbers shouldn't be any work.
            $endgroup$
            – Martin Kochanski
            May 21 at 17:57










          • $begingroup$
            @MartinKochanski I'll get back to you tomorrow.
            $endgroup$
            – Oldboy
            May 21 at 19:45










          • $begingroup$
            Thanks: I have just done some more working out, and I can see where my recurrence went wrong. So don't bother with any more simulations after all.
            $endgroup$
            – Martin Kochanski
            May 21 at 20:01
















          • $begingroup$
            Splendid. There is just one snag (or one advantage!): these figures contradict mine quite strongly. I can't verifiably take my recurrence as far up as your numbers, so I wonder: could you let me know what (a) your average maximum arm length and (b) your commonest maximum arm length are for each of 1 to 9 pairs? With any luck we will get a clear enough and noticeable enough deviation for me to identify where I've been going wrong. I am only asking because you have already written the program and plugging in some small numbers shouldn't be any work.
            $endgroup$
            – Martin Kochanski
            May 21 at 17:57










          • $begingroup$
            @MartinKochanski I'll get back to you tomorrow.
            $endgroup$
            – Oldboy
            May 21 at 19:45










          • $begingroup$
            Thanks: I have just done some more working out, and I can see where my recurrence went wrong. So don't bother with any more simulations after all.
            $endgroup$
            – Martin Kochanski
            May 21 at 20:01















          $begingroup$
          Splendid. There is just one snag (or one advantage!): these figures contradict mine quite strongly. I can't verifiably take my recurrence as far up as your numbers, so I wonder: could you let me know what (a) your average maximum arm length and (b) your commonest maximum arm length are for each of 1 to 9 pairs? With any luck we will get a clear enough and noticeable enough deviation for me to identify where I've been going wrong. I am only asking because you have already written the program and plugging in some small numbers shouldn't be any work.
          $endgroup$
          – Martin Kochanski
          May 21 at 17:57




          $begingroup$
          Splendid. There is just one snag (or one advantage!): these figures contradict mine quite strongly. I can't verifiably take my recurrence as far up as your numbers, so I wonder: could you let me know what (a) your average maximum arm length and (b) your commonest maximum arm length are for each of 1 to 9 pairs? With any luck we will get a clear enough and noticeable enough deviation for me to identify where I've been going wrong. I am only asking because you have already written the program and plugging in some small numbers shouldn't be any work.
          $endgroup$
          – Martin Kochanski
          May 21 at 17:57












          $begingroup$
          @MartinKochanski I'll get back to you tomorrow.
          $endgroup$
          – Oldboy
          May 21 at 19:45




          $begingroup$
          @MartinKochanski I'll get back to you tomorrow.
          $endgroup$
          – Oldboy
          May 21 at 19:45












          $begingroup$
          Thanks: I have just done some more working out, and I can see where my recurrence went wrong. So don't bother with any more simulations after all.
          $endgroup$
          – Martin Kochanski
          May 21 at 20:01




          $begingroup$
          Thanks: I have just done some more working out, and I can see where my recurrence went wrong. So don't bother with any more simulations after all.
          $endgroup$
          – Martin Kochanski
          May 21 at 20:01











          14












          $begingroup$

          The expected number of single socks is maximized when you are halfway through. When you have drawn $N$ socks the chance that a given pair has one on your arm is $frac 2N^22N^2+2N(N-1)=fracN^22N^2-Napprox frac 12+frac 12N$. If we make the socks distinguishable, to have one on your arm sock $1$ of a pair has $2N$ positions it can be in, then sock $2$ has $N$ choices-to be in the other half of the run. To not have one on your arm sock $1$ again has $2N$ choices but sock $2$ has only $N-1$ as it must be in the same half of the run. This says the expected number on your arm is $frac N^22N-1approx frac N+12$.



          The expected value being below the mode of Oldboy's distributions says that the distribution is not symmetric around the mode.



          Note that this addresses the expected maximum at a given point. The expected maximum over a distribution can be higher as Empy2 explains.






          share|cite|improve this answer











          $endgroup$












          • $begingroup$
            Could you please explain why when you have drawn $N$ socks the chance that a given pair has one on your arm is $frac N^2N^2+N(N-1)$?
            $endgroup$
            – Hans
            May 23 at 5:47










          • $begingroup$
            @Hans: I said the expected value of the number of socks on your arm was that. I computed the chance you have one of a particular pair on your arm, then multiplied by $N$ using the linearity of expectation to get the total expectation.
            $endgroup$
            – Ross Millikan
            May 23 at 14:36











          • $begingroup$
            I am confused. I just copied your claim: "When you have drawn $N$ socks the chance that a given pair has one on your arm is $frac N^2N^2+N(N-1)$." Are you saying you do not mean this expression to be a probability but an expectation? If you do mean this to be a probability, I do not understand your rationale for this expression. In fact, I got a different expression. Could you please derive this expression of the probability?
            $endgroup$
            – Hans
            May 23 at 17:51










          • $begingroup$
            @Hans: For one pair, the probability and the expectation are the same because you either have one sock on your arm or not. I justify the probability in the next sentence, though I left out a factor $2$. There are $2N^2$ ways to choose the positions of the socks of one pair so you have one on your arm at the midpoint and $2N(N-1)$ ways to not have one on your arm.
            $endgroup$
            – Ross Millikan
            May 23 at 19:35










          • $begingroup$
            You are right. +1. I took the individual socks of a pair as indistinguishable which was wrong.
            $endgroup$
            – Hans
            2 days ago















          14












          $begingroup$

          The expected number of single socks is maximized when you are halfway through. When you have drawn $N$ socks the chance that a given pair has one on your arm is $frac 2N^22N^2+2N(N-1)=fracN^22N^2-Napprox frac 12+frac 12N$. If we make the socks distinguishable, to have one on your arm sock $1$ of a pair has $2N$ positions it can be in, then sock $2$ has $N$ choices-to be in the other half of the run. To not have one on your arm sock $1$ again has $2N$ choices but sock $2$ has only $N-1$ as it must be in the same half of the run. This says the expected number on your arm is $frac N^22N-1approx frac N+12$.



          The expected value being below the mode of Oldboy's distributions says that the distribution is not symmetric around the mode.



          Note that this addresses the expected maximum at a given point. The expected maximum over a distribution can be higher as Empy2 explains.






          share|cite|improve this answer











          $endgroup$












          • $begingroup$
            Could you please explain why when you have drawn $N$ socks the chance that a given pair has one on your arm is $frac N^2N^2+N(N-1)$?
            $endgroup$
            – Hans
            May 23 at 5:47










          • $begingroup$
            @Hans: I said the expected value of the number of socks on your arm was that. I computed the chance you have one of a particular pair on your arm, then multiplied by $N$ using the linearity of expectation to get the total expectation.
            $endgroup$
            – Ross Millikan
            May 23 at 14:36











          • $begingroup$
            I am confused. I just copied your claim: "When you have drawn $N$ socks the chance that a given pair has one on your arm is $frac N^2N^2+N(N-1)$." Are you saying you do not mean this expression to be a probability but an expectation? If you do mean this to be a probability, I do not understand your rationale for this expression. In fact, I got a different expression. Could you please derive this expression of the probability?
            $endgroup$
            – Hans
            May 23 at 17:51










          • $begingroup$
            @Hans: For one pair, the probability and the expectation are the same because you either have one sock on your arm or not. I justify the probability in the next sentence, though I left out a factor $2$. There are $2N^2$ ways to choose the positions of the socks of one pair so you have one on your arm at the midpoint and $2N(N-1)$ ways to not have one on your arm.
            $endgroup$
            – Ross Millikan
            May 23 at 19:35










          • $begingroup$
            You are right. +1. I took the individual socks of a pair as indistinguishable which was wrong.
            $endgroup$
            – Hans
            2 days ago













          14












          14








          14





          $begingroup$

          The expected number of single socks is maximized when you are halfway through. When you have drawn $N$ socks the chance that a given pair has one on your arm is $frac 2N^22N^2+2N(N-1)=fracN^22N^2-Napprox frac 12+frac 12N$. If we make the socks distinguishable, to have one on your arm sock $1$ of a pair has $2N$ positions it can be in, then sock $2$ has $N$ choices-to be in the other half of the run. To not have one on your arm sock $1$ again has $2N$ choices but sock $2$ has only $N-1$ as it must be in the same half of the run. This says the expected number on your arm is $frac N^22N-1approx frac N+12$.



          The expected value being below the mode of Oldboy's distributions says that the distribution is not symmetric around the mode.



          Note that this addresses the expected maximum at a given point. The expected maximum over a distribution can be higher as Empy2 explains.






          share|cite|improve this answer











          $endgroup$



          The expected number of single socks is maximized when you are halfway through. When you have drawn $N$ socks the chance that a given pair has one on your arm is $frac 2N^22N^2+2N(N-1)=fracN^22N^2-Napprox frac 12+frac 12N$. If we make the socks distinguishable, to have one on your arm sock $1$ of a pair has $2N$ positions it can be in, then sock $2$ has $N$ choices-to be in the other half of the run. To not have one on your arm sock $1$ again has $2N$ choices but sock $2$ has only $N-1$ as it must be in the same half of the run. This says the expected number on your arm is $frac N^22N-1approx frac N+12$.



          The expected value being below the mode of Oldboy's distributions says that the distribution is not symmetric around the mode.



          Note that this addresses the expected maximum at a given point. The expected maximum over a distribution can be higher as Empy2 explains.







          share|cite|improve this answer














          share|cite|improve this answer



          share|cite|improve this answer








          edited May 23 at 19:35

























          answered May 21 at 13:54









          Ross MillikanRoss Millikan

          305k24202378




          305k24202378











          • $begingroup$
            Could you please explain why when you have drawn $N$ socks the chance that a given pair has one on your arm is $frac N^2N^2+N(N-1)$?
            $endgroup$
            – Hans
            May 23 at 5:47










          • $begingroup$
            @Hans: I said the expected value of the number of socks on your arm was that. I computed the chance you have one of a particular pair on your arm, then multiplied by $N$ using the linearity of expectation to get the total expectation.
            $endgroup$
            – Ross Millikan
            May 23 at 14:36











          • $begingroup$
            I am confused. I just copied your claim: "When you have drawn $N$ socks the chance that a given pair has one on your arm is $frac N^2N^2+N(N-1)$." Are you saying you do not mean this expression to be a probability but an expectation? If you do mean this to be a probability, I do not understand your rationale for this expression. In fact, I got a different expression. Could you please derive this expression of the probability?
            $endgroup$
            – Hans
            May 23 at 17:51










          • $begingroup$
            @Hans: For one pair, the probability and the expectation are the same because you either have one sock on your arm or not. I justify the probability in the next sentence, though I left out a factor $2$. There are $2N^2$ ways to choose the positions of the socks of one pair so you have one on your arm at the midpoint and $2N(N-1)$ ways to not have one on your arm.
            $endgroup$
            – Ross Millikan
            May 23 at 19:35










          • $begingroup$
            You are right. +1. I took the individual socks of a pair as indistinguishable which was wrong.
            $endgroup$
            – Hans
            2 days ago
















          • $begingroup$
            Could you please explain why when you have drawn $N$ socks the chance that a given pair has one on your arm is $frac N^2N^2+N(N-1)$?
            $endgroup$
            – Hans
            May 23 at 5:47










          • $begingroup$
            @Hans: I said the expected value of the number of socks on your arm was that. I computed the chance you have one of a particular pair on your arm, then multiplied by $N$ using the linearity of expectation to get the total expectation.
            $endgroup$
            – Ross Millikan
            May 23 at 14:36











          • $begingroup$
            I am confused. I just copied your claim: "When you have drawn $N$ socks the chance that a given pair has one on your arm is $frac N^2N^2+N(N-1)$." Are you saying you do not mean this expression to be a probability but an expectation? If you do mean this to be a probability, I do not understand your rationale for this expression. In fact, I got a different expression. Could you please derive this expression of the probability?
            $endgroup$
            – Hans
            May 23 at 17:51










          • $begingroup$
            @Hans: For one pair, the probability and the expectation are the same because you either have one sock on your arm or not. I justify the probability in the next sentence, though I left out a factor $2$. There are $2N^2$ ways to choose the positions of the socks of one pair so you have one on your arm at the midpoint and $2N(N-1)$ ways to not have one on your arm.
            $endgroup$
            – Ross Millikan
            May 23 at 19:35










          • $begingroup$
            You are right. +1. I took the individual socks of a pair as indistinguishable which was wrong.
            $endgroup$
            – Hans
            2 days ago















          $begingroup$
          Could you please explain why when you have drawn $N$ socks the chance that a given pair has one on your arm is $frac N^2N^2+N(N-1)$?
          $endgroup$
          – Hans
          May 23 at 5:47




          $begingroup$
          Could you please explain why when you have drawn $N$ socks the chance that a given pair has one on your arm is $frac N^2N^2+N(N-1)$?
          $endgroup$
          – Hans
          May 23 at 5:47












          $begingroup$
          @Hans: I said the expected value of the number of socks on your arm was that. I computed the chance you have one of a particular pair on your arm, then multiplied by $N$ using the linearity of expectation to get the total expectation.
          $endgroup$
          – Ross Millikan
          May 23 at 14:36





          $begingroup$
          @Hans: I said the expected value of the number of socks on your arm was that. I computed the chance you have one of a particular pair on your arm, then multiplied by $N$ using the linearity of expectation to get the total expectation.
          $endgroup$
          – Ross Millikan
          May 23 at 14:36













          $begingroup$
          I am confused. I just copied your claim: "When you have drawn $N$ socks the chance that a given pair has one on your arm is $frac N^2N^2+N(N-1)$." Are you saying you do not mean this expression to be a probability but an expectation? If you do mean this to be a probability, I do not understand your rationale for this expression. In fact, I got a different expression. Could you please derive this expression of the probability?
          $endgroup$
          – Hans
          May 23 at 17:51




          $begingroup$
          I am confused. I just copied your claim: "When you have drawn $N$ socks the chance that a given pair has one on your arm is $frac N^2N^2+N(N-1)$." Are you saying you do not mean this expression to be a probability but an expectation? If you do mean this to be a probability, I do not understand your rationale for this expression. In fact, I got a different expression. Could you please derive this expression of the probability?
          $endgroup$
          – Hans
          May 23 at 17:51












          $begingroup$
          @Hans: For one pair, the probability and the expectation are the same because you either have one sock on your arm or not. I justify the probability in the next sentence, though I left out a factor $2$. There are $2N^2$ ways to choose the positions of the socks of one pair so you have one on your arm at the midpoint and $2N(N-1)$ ways to not have one on your arm.
          $endgroup$
          – Ross Millikan
          May 23 at 19:35




          $begingroup$
          @Hans: For one pair, the probability and the expectation are the same because you either have one sock on your arm or not. I justify the probability in the next sentence, though I left out a factor $2$. There are $2N^2$ ways to choose the positions of the socks of one pair so you have one on your arm at the midpoint and $2N(N-1)$ ways to not have one on your arm.
          $endgroup$
          – Ross Millikan
          May 23 at 19:35












          $begingroup$
          You are right. +1. I took the individual socks of a pair as indistinguishable which was wrong.
          $endgroup$
          – Hans
          2 days ago




          $begingroup$
          You are right. +1. I took the individual socks of a pair as indistinguishable which was wrong.
          $endgroup$
          – Hans
          2 days ago











          8












          $begingroup$

          The extra bits, above $N/2$ in Oldboy's table, are near $sqrt[3]N$. I have some ideas why that might be true.



          First, the expected number of socks on the arm at the $N+x$th sock is $(N^2-x^2)/(2N-1)$.



          Near the $N$th sock, the number of socks on the arm follows a random walk. It is symmetric at the $N$th sock, but has a negative bias of $x^2/2N$ at the $N+x$th sock.

          Around the $N+yN^2/3$th sock, a symmetric random walk would have moved $O(sqrtyN^2/3)$, but the negative bias is also $O((yN^2/3)^2/N)$, so both are $O(N^1/3)$. The negative bias will dominate for large $y$, so the maximum value will be in this domain. The random variation dominates for small $y$.



          So the maximum is likely to be $N/2+O(N^1/3)$.



          EDIT: the variance in the number of socks on the arm at the $N$th sock is $$frac2N^2(N-1)^2(2N-1)^2(2N-3)approxfrac N4$$
          So the width of the bell curve in Oldboy's graphs is roughly $sqrtN$. But this effect is symmetric above and below the mean $N^2/(2N-1)$. The maximum of the random walk is not symmetric, and shifts the bell curve to the right, but that effect $(O(N^1/3))$ is smaller than the variation from one laundry basket to the next $(O(N^1/2))$






          share|cite|improve this answer











          $endgroup$












          • $begingroup$
            +1 Excellent obeservation and it fits my data almost perfectly.
            $endgroup$
            – Oldboy
            May 22 at 6:59










          • $begingroup$
            The denominator of the variance approximation of the number of socks on the arm should be $4$ not $2$.
            $endgroup$
            – Hans
            7 hours ago










          • $begingroup$
            1) By the random walk of the number of socks on the arm "has a negative bias", do you simply mean the difference between the mean and $frac N2$ which is $-fracx^22N-1$? 2) Could you please explain your statement "a symmetric random walk would have moved $O(sqrtyN^2/3)$"? Should the standard deviation be instead $sqrtN+yN^2/3$?
            $endgroup$
            – Hans
            2 hours ago











          • $begingroup$
            I meant that the middle, at $N$, is expected to be the highest point, butas you wander away from there the is a random walk that goes above and below, so the maximum is likely to be above the centre point. It will probably only get $O(N^1/3)$ above before the mean drags it back below the centre point
            $endgroup$
            – Empy2
            2 hours ago















          8












          $begingroup$

          The extra bits, above $N/2$ in Oldboy's table, are near $sqrt[3]N$. I have some ideas why that might be true.



          First, the expected number of socks on the arm at the $N+x$th sock is $(N^2-x^2)/(2N-1)$.



          Near the $N$th sock, the number of socks on the arm follows a random walk. It is symmetric at the $N$th sock, but has a negative bias of $x^2/2N$ at the $N+x$th sock.

          Around the $N+yN^2/3$th sock, a symmetric random walk would have moved $O(sqrtyN^2/3)$, but the negative bias is also $O((yN^2/3)^2/N)$, so both are $O(N^1/3)$. The negative bias will dominate for large $y$, so the maximum value will be in this domain. The random variation dominates for small $y$.



          So the maximum is likely to be $N/2+O(N^1/3)$.



          EDIT: the variance in the number of socks on the arm at the $N$th sock is $$frac2N^2(N-1)^2(2N-1)^2(2N-3)approxfrac N4$$
          So the width of the bell curve in Oldboy's graphs is roughly $sqrtN$. But this effect is symmetric above and below the mean $N^2/(2N-1)$. The maximum of the random walk is not symmetric, and shifts the bell curve to the right, but that effect $(O(N^1/3))$ is smaller than the variation from one laundry basket to the next $(O(N^1/2))$






          share|cite|improve this answer











          $endgroup$












          • $begingroup$
            +1 Excellent obeservation and it fits my data almost perfectly.
            $endgroup$
            – Oldboy
            May 22 at 6:59










          • $begingroup$
            The denominator of the variance approximation of the number of socks on the arm should be $4$ not $2$.
            $endgroup$
            – Hans
            7 hours ago










          • $begingroup$
            1) By the random walk of the number of socks on the arm "has a negative bias", do you simply mean the difference between the mean and $frac N2$ which is $-fracx^22N-1$? 2) Could you please explain your statement "a symmetric random walk would have moved $O(sqrtyN^2/3)$"? Should the standard deviation be instead $sqrtN+yN^2/3$?
            $endgroup$
            – Hans
            2 hours ago











          • $begingroup$
            I meant that the middle, at $N$, is expected to be the highest point, butas you wander away from there the is a random walk that goes above and below, so the maximum is likely to be above the centre point. It will probably only get $O(N^1/3)$ above before the mean drags it back below the centre point
            $endgroup$
            – Empy2
            2 hours ago













          8












          8








          8





          $begingroup$

          The extra bits, above $N/2$ in Oldboy's table, are near $sqrt[3]N$. I have some ideas why that might be true.



          First, the expected number of socks on the arm at the $N+x$th sock is $(N^2-x^2)/(2N-1)$.



          Near the $N$th sock, the number of socks on the arm follows a random walk. It is symmetric at the $N$th sock, but has a negative bias of $x^2/2N$ at the $N+x$th sock.

          Around the $N+yN^2/3$th sock, a symmetric random walk would have moved $O(sqrtyN^2/3)$, but the negative bias is also $O((yN^2/3)^2/N)$, so both are $O(N^1/3)$. The negative bias will dominate for large $y$, so the maximum value will be in this domain. The random variation dominates for small $y$.



          So the maximum is likely to be $N/2+O(N^1/3)$.



          EDIT: the variance in the number of socks on the arm at the $N$th sock is $$frac2N^2(N-1)^2(2N-1)^2(2N-3)approxfrac N4$$
          So the width of the bell curve in Oldboy's graphs is roughly $sqrtN$. But this effect is symmetric above and below the mean $N^2/(2N-1)$. The maximum of the random walk is not symmetric, and shifts the bell curve to the right, but that effect $(O(N^1/3))$ is smaller than the variation from one laundry basket to the next $(O(N^1/2))$






          share|cite|improve this answer











          $endgroup$



          The extra bits, above $N/2$ in Oldboy's table, are near $sqrt[3]N$. I have some ideas why that might be true.



          First, the expected number of socks on the arm at the $N+x$th sock is $(N^2-x^2)/(2N-1)$.



          Near the $N$th sock, the number of socks on the arm follows a random walk. It is symmetric at the $N$th sock, but has a negative bias of $x^2/2N$ at the $N+x$th sock.

          Around the $N+yN^2/3$th sock, a symmetric random walk would have moved $O(sqrtyN^2/3)$, but the negative bias is also $O((yN^2/3)^2/N)$, so both are $O(N^1/3)$. The negative bias will dominate for large $y$, so the maximum value will be in this domain. The random variation dominates for small $y$.



          So the maximum is likely to be $N/2+O(N^1/3)$.



          EDIT: the variance in the number of socks on the arm at the $N$th sock is $$frac2N^2(N-1)^2(2N-1)^2(2N-3)approxfrac N4$$
          So the width of the bell curve in Oldboy's graphs is roughly $sqrtN$. But this effect is symmetric above and below the mean $N^2/(2N-1)$. The maximum of the random walk is not symmetric, and shifts the bell curve to the right, but that effect $(O(N^1/3))$ is smaller than the variation from one laundry basket to the next $(O(N^1/2))$







          share|cite|improve this answer














          share|cite|improve this answer



          share|cite|improve this answer








          edited 7 hours ago

























          answered May 21 at 16:12









          Empy2Empy2

          34.2k12562




          34.2k12562











          • $begingroup$
            +1 Excellent obeservation and it fits my data almost perfectly.
            $endgroup$
            – Oldboy
            May 22 at 6:59










          • $begingroup$
            The denominator of the variance approximation of the number of socks on the arm should be $4$ not $2$.
            $endgroup$
            – Hans
            7 hours ago










          • $begingroup$
            1) By the random walk of the number of socks on the arm "has a negative bias", do you simply mean the difference between the mean and $frac N2$ which is $-fracx^22N-1$? 2) Could you please explain your statement "a symmetric random walk would have moved $O(sqrtyN^2/3)$"? Should the standard deviation be instead $sqrtN+yN^2/3$?
            $endgroup$
            – Hans
            2 hours ago











          • $begingroup$
            I meant that the middle, at $N$, is expected to be the highest point, butas you wander away from there the is a random walk that goes above and below, so the maximum is likely to be above the centre point. It will probably only get $O(N^1/3)$ above before the mean drags it back below the centre point
            $endgroup$
            – Empy2
            2 hours ago
















          • $begingroup$
            +1 Excellent obeservation and it fits my data almost perfectly.
            $endgroup$
            – Oldboy
            May 22 at 6:59










          • $begingroup$
            The denominator of the variance approximation of the number of socks on the arm should be $4$ not $2$.
            $endgroup$
            – Hans
            7 hours ago










          • $begingroup$
            1) By the random walk of the number of socks on the arm "has a negative bias", do you simply mean the difference between the mean and $frac N2$ which is $-fracx^22N-1$? 2) Could you please explain your statement "a symmetric random walk would have moved $O(sqrtyN^2/3)$"? Should the standard deviation be instead $sqrtN+yN^2/3$?
            $endgroup$
            – Hans
            2 hours ago











          • $begingroup$
            I meant that the middle, at $N$, is expected to be the highest point, butas you wander away from there the is a random walk that goes above and below, so the maximum is likely to be above the centre point. It will probably only get $O(N^1/3)$ above before the mean drags it back below the centre point
            $endgroup$
            – Empy2
            2 hours ago















          $begingroup$
          +1 Excellent obeservation and it fits my data almost perfectly.
          $endgroup$
          – Oldboy
          May 22 at 6:59




          $begingroup$
          +1 Excellent obeservation and it fits my data almost perfectly.
          $endgroup$
          – Oldboy
          May 22 at 6:59












          $begingroup$
          The denominator of the variance approximation of the number of socks on the arm should be $4$ not $2$.
          $endgroup$
          – Hans
          7 hours ago




          $begingroup$
          The denominator of the variance approximation of the number of socks on the arm should be $4$ not $2$.
          $endgroup$
          – Hans
          7 hours ago












          $begingroup$
          1) By the random walk of the number of socks on the arm "has a negative bias", do you simply mean the difference between the mean and $frac N2$ which is $-fracx^22N-1$? 2) Could you please explain your statement "a symmetric random walk would have moved $O(sqrtyN^2/3)$"? Should the standard deviation be instead $sqrtN+yN^2/3$?
          $endgroup$
          – Hans
          2 hours ago





          $begingroup$
          1) By the random walk of the number of socks on the arm "has a negative bias", do you simply mean the difference between the mean and $frac N2$ which is $-fracx^22N-1$? 2) Could you please explain your statement "a symmetric random walk would have moved $O(sqrtyN^2/3)$"? Should the standard deviation be instead $sqrtN+yN^2/3$?
          $endgroup$
          – Hans
          2 hours ago













          $begingroup$
          I meant that the middle, at $N$, is expected to be the highest point, butas you wander away from there the is a random walk that goes above and below, so the maximum is likely to be above the centre point. It will probably only get $O(N^1/3)$ above before the mean drags it back below the centre point
          $endgroup$
          – Empy2
          2 hours ago




          $begingroup$
          I meant that the middle, at $N$, is expected to be the highest point, butas you wander away from there the is a random walk that goes above and below, so the maximum is likely to be above the centre point. It will probably only get $O(N^1/3)$ above before the mean drags it back below the centre point
          $endgroup$
          – Empy2
          2 hours ago

















          draft saved

          draft discarded
















































          Thanks for contributing an answer to Mathematics Stack Exchange!


          • Please be sure to answer the question. Provide details and share your research!

          But avoid


          • Asking for help, clarification, or responding to other answers.

          • Making statements based on opinion; back them up with references or personal experience.

          Use MathJax to format equations. MathJax reference.


          To learn more, see our tips on writing great answers.




          draft saved


          draft discarded














          StackExchange.ready(
          function ()
          StackExchange.openid.initPostLogin('.new-post-login', 'https%3a%2f%2fmath.stackexchange.com%2fquestions%2f3234157%2fexpected-maximum-number-of-unpaired-socks%23new-answer', 'question_page');

          );

          Post as a guest















          Required, but never shown





















































          Required, but never shown














          Required, but never shown












          Required, but never shown







          Required, but never shown

































          Required, but never shown














          Required, but never shown












          Required, but never shown







          Required, but never shown







          Popular posts from this blog

          Get product attribute by attribute group code in magento 2get product attribute by product attribute group in magento 2Magento 2 Log Bundle Product Data in List Page?How to get all product attribute of a attribute group of Default attribute set?Magento 2.1 Create a filter in the product grid by new attributeMagento 2 : Get Product Attribute values By GroupMagento 2 How to get all existing values for one attributeMagento 2 get custom attribute of a single product inside a pluginMagento 2.3 How to get all the Multi Source Inventory (MSI) locations collection in custom module?Magento2: how to develop rest API to get new productsGet product attribute by attribute group code ( [attribute_group_code] ) in magento 2

          Category:9 (number) SubcategoriesMedia in category "9 (number)"Navigation menuUpload mediaGND ID: 4485639-8Library of Congress authority ID: sh85091979ReasonatorScholiaStatistics

          Magento 2.3: How do i solve this, Not registered handle, on custom form?How can i rewrite TierPrice Block in Magento2magento 2 captcha not rendering if I override layout xmlmain.CRITICAL: Plugin class doesn't existMagento 2 : Problem while adding custom button order view page?Magento 2.2.5: Overriding Admin Controller sales/orderMagento 2.2.5: Add, Update and Delete existing products Custom OptionsMagento 2.3 : File Upload issue in UI Component FormMagento2 Not registered handleHow to configured Form Builder Js in my custom magento 2.3.0 module?Magento 2.3. How to create image upload field in an admin form